Jump to content

chardhdmovies's Content

There have been 621 items by chardhdmovies (Search limited from 08-06-2020)



Sort by                Order  

#636267 CMR số cách chọn các nhóm như thế là số lẻ

Posted by chardhdmovies on 28-05-2016 - 15:50 in Tổ hợp và rời rạc

Cho $229$ học sinh nam và $271$ học sinh nữ được chia thành $10$ nhóm,mỗi nhóm gồm $50$ người được đánh số từ $1$ đến $50$.Người ta muốn chọn ra một nhóm $4$ học sinh trong đó số học sinh nữ được chọn là số lẻ và thỏa mãn điều kiện sau:$4$ người này được chọn từ $2$ nhóm và có $2$ cặp học sinh có cùng số thứ tự,Chứng minh rằng số cách chọn các nhóm như thế là số lẻ




#616177 Tìm số cách tô màu thỏa đề.

Posted by chardhdmovies on 21-02-2016 - 09:00 in Tổ hợp và rời rạc

Cho đa giác đều $n$ đỉnh nội tiếp đường tròn tâm $O$, tìm số cách tô $n$ đỉnh tam giác bằng $m (m\geq2)$ màu sao cho 2 đỉnh kề nhau không được tô cùng màu. (Hai cách tô được coi là giống nhau nếu chúng nhận được qua nhau bằng một phép quay tâm $O$)




#614806 $\frac{x^4}{(x^2+y^2)(x+y)}$

Posted by chardhdmovies on 13-02-2016 - 23:16 in Bất đẳng thức và cực trị

Cho $x,y,z>0$ thoả mãn $x+y+z=1$. Tìm $min$ của $F=\frac{x^4}{(x^2+y^2)(x+y)}+\frac{y^4}{(y^2+z^2)(y+z)}+\frac{z^4}{(z^2+x^2)(z+x)}$
tks you =))

Gợi ý: Chứng minh $\sum{\frac{x^4}{(x^2+y^2)(x+y)}}=\sum{\frac{y^4}{(x^2+y^2)(x+y)}}$




#607050 Chứng minh rằng $[\frac{n}{3}]+[\frac...

Posted by chardhdmovies on 03-01-2016 - 21:22 in Số học

Cho $n$ là số tự nhiên. Kí hiệu $\left [ n \right ]$ là số nguyên lớn nhất không lớn hơn $n$. Chứng minh:
$\left [ \frac{n}{3} \right ]+\left [ \frac{n}{35} \right ]\geq \left [ \frac{n}{5} \right ]+\left [ \frac{n}{7} \right ]$




#606966 TOPIC ôn thi Olimpic 30/04 và thi HSG toán 10

Posted by chardhdmovies on 03-01-2016 - 15:46 in Thi HSG cấp Tỉnh, Thành phố. Olympic 30-4. Đề thi và kiểm tra đội tuyển các cấp.

Nhiệt liệt ủng hộ topic này luôn, chúc topic phát triển và đóng góp nhiều bài tập hay  :lol:

Bài 2: Cho a, b, c, d là các số thực dương thỏa mãn đk: $\frac{1}{a} + \frac{1}{b} + \frac{1}{c} + \frac{1}{d} = 4$. CM:

$\sqrt[3]{\frac{a^{3} + b^{3}}{2}} + \sqrt[3]{\frac{b^{3} + c^{3}}{2}} + \sqrt[3]{\frac{c^{3} + d^{3}}{2}} + \sqrt[3]{\frac{d^{3} + a^{3}}{2}} \leq 2(a + b + c + d) - 4$

Bài 3: CMR nếu a,b,c>0 thì: 

$\frac{(a + b + c)^{2}}{ab + bc + ca} \geq \frac{a + b}{a + c} + \frac{b + c}{b + a} + \frac{c + a}{c + b}$

3. Giả sử $c=min\left \{ a,b,c \right \}$. Ta biến đổi BĐT như sau:

$\frac{(a+b+c)^{2}}{ab+bc+ca}-3\geq (\frac{a+b}{a+c}-1)+(\frac{b+c}{b+a}-1)+(\frac{c+a}{c+b}-1)$

$\Leftrightarrow \frac{a^2+b^2+c^2-ab-bc-ca}{ab+bc+ca}\geq \frac{b-c}{a+c}+\frac{c-a}{b+a}+\frac{a-b}{c+b}$

$\Leftrightarrow \frac{(a-b)^2+(a-c)(b-c)}{ab+bc+ca}\geq\frac{(a-c)(b-c)+b^2-a^2}{(a+c)(a+b)}+\frac{a-b}{b+c}$
$\Leftrightarrow \frac{(a-b)^2+(a-c)(b-c)}{ab+bc+ca}\geq\frac{(a-c)(b-c)}{(a+c)(a+b)}+\frac{(a-b)^2}{(a+c)(b+c)}$

$\Leftrightarrow \frac{c^{2}(a-b)^2}{(ab+bc+ca)(a+c)(b+c)}+\frac{a^2(a-c)(b-c)}{(ab+bc+ca)(a+c)(a+b)}\geq0$
BĐT cuối cùng luôn đúng do $c=min\left \{ a,b,c \right \}$

Đẳng thức xảy ra $\Leftrightarrow a=b=c$




#606958 Chứng minh $f(x)\neq 0,\forall x \in \mathbb{R...

Posted by chardhdmovies on 03-01-2016 - 14:54 in Phương trình hàm

Chứng ta còn có thể tìm ra $f(x)$ luôn kìa bạn, chứng minh cũng khá dài

Và bài này là 1 bổ đề quan trọng để tìm hàm $f$

Mình xin chứng minh bổ đề này

Thay $x=0,y=0 => f^4(0)=f^2(0) => f(0)=0 ; f^2(0)=1$

TH1: $f(0)=0$, thay $y=0 => f(2x) =0 => f(x)=0 $ ( vô lý )

TH2: $f^2(0)=1$

Giả sử $\exists x_0: f(x_0)=0 $

Ta có $f(2x).f(0)=f^4(x)=> f(x).f(0)=f^4(\frac{x}{2})$

Thay $x \rightarrow x_0 => f^4(\frac{x_0}{2}) =0 => f(\frac{x_0}{2})=0$

Đẩy liên tục $=> f(\frac{x_0}{2^k})=0 $

Cho k cực lớn $=> f(0)=0 $ (vô lý )

Do đó $f(x) \neq 0, \forall x \in \mathbb{R} $

Mình còn thiếu bước này để tìm f, ngay sau khi đăng topic mình cũng đã cm được. Dù sao cũng cảm ơn bạn.




#606704 Chứng minh $f(x)\neq 0,\forall x \in \mathbb{R...

Posted by chardhdmovies on 02-01-2016 - 12:23 in Phương trình hàm

Cho hàm $f:\mathbb{R} \rightarrow \mathbb{R}$ liên tục và khác hàm hằng thỏa:

$f(x+y)f(x-y)=(f(x)f(y))^2, \forall x \in \mathbb{R}$

Chứng minh: $f(x)\neq 0,\forall x \in \mathbb{R}$




#606445 Chứng minh tứ giác AFCD nội tiếp.

Posted by chardhdmovies on 01-01-2016 - 12:49 in Hình học

Hint: $\bullet$ để ý $\Delta ANB=\Delta END$ .... một hồi ta có $\frac{NA}{ND}=\frac{EF}{BC}=\frac{NF}{NC}$ từ đó có $AFCD$ nội tiếp đường tròn tâm $I$

         $\bullet$ để chứng minh ý sau ta chứng minh $NO_1IO_2$ là hình bình hành

Spoiler

 vậy là đủ rồi  :D




#606415 Chứng minh tứ giác AFCD nội tiếp.

Posted by chardhdmovies on 01-01-2016 - 09:25 in Hình học

Cho hai đường tròn $(O_1),(O_2)$ cắt nhau tại hai điểm $M,N$. Qua $A$ thuộc $(O_1)$ kẻ các đường thẳng $AM,AN$, lần lượt cắt $(O_2)$ tại $B, C$. Qua $D$ thuộc $(O_2)$ kẻ các đường thẳng $DM,DN$, lần lượt cắt $(O_1)$ tại $E, F$. Các điểm $A,E,F$ cùng nằm về một phía đối với đường thẳng $MN$, các điểm $D,B,C$ cùng nằm về phía khác. Chứng minh nếu $AB=DE$ thì 4 điểm $A,F,C,D$ cùng nằm trên một đường tròn có tâm không phụ thuộc vào vị trí của $A$ và $D$.

P/s: Mình nghĩ tâm là giao điểm của $(O_{1}O_{2}M)$ và đường vuông góc với $MN$ tại $M$.




#606203 $[\sum a^2+k\sum ab][\sum \frac{1}{(a...

Posted by chardhdmovies on 30-12-2015 - 21:35 in Bất đẳng thức - Cực trị

Lời giải của mình cũng dựa trên ý tưởng trên nhưng dài hơn.

Giả sử $a>c>b$, đặt vế trái là $f(a,b,c)$.

TH1: $ab+bc+ca\geq(a-c)(b-c)$, dễ thấy $f(a,b,c)\geq f(a-c,b-c,0)$

Cần chứng minh: $f(a-c,b-c,0)\geq0$

Đặt $a-c=x>0,c-b=y>0$, bđt cần chứng minh trở thành.

$[x^2+y^2-kxy][\frac{1}{x^2}+\frac{1}{y^2}+\frac{1}{(x+y)^2}]\geq \frac{9(2-k)}{4}$

$VT \geq[(x+y)^2-\frac{(k+2)(x+y)^2}{4}][\frac{8}{(x+y)^2}+\frac{1}{(x+y)^2}]=VP$

TH2: $ab+bc+ca\leq(a-c)(b-c)$

$a^2+b^2+c^2+k(ab+bc+ca)=(a+b+c)^2+(k-2)(ab+bc+ca)\geq(k-2)(a-c)(b-c)$

Cần chứng minh: $[(a-c)(c-b)][\frac{1}{(a-b)^2}+\frac{1}{(b-c)^2}+\frac{1}{(c-a)^2}\geq\frac{9}{4}$

Đặt $a-c=m>0,c-b=n>0$, bđt cần chứng minh trở thành.

$mn[\frac{1}{m^2}+\frac{1}{n^2}+\frac{1}{(m+n)^2}]\geq\frac{9}{4} \Leftrightarrow \frac{(m-n)^2(m^2+mn+n^2)}{(m+n)^2}\geq0$




#606148 $[\sum a^2+k\sum ab][\sum \frac{1}{(a...

Posted by chardhdmovies on 30-12-2015 - 18:49 in Bất đẳng thức - Cực trị

Cho số thực $k\in [-1,2]$, chứng minh BĐT sau đúng với mọi số thực $a,b,c$:
$[a^2+b^2+c^2+k(ab+bc+ca)][\frac{1}{(a-b)^{2}}+\frac{1}{(b-c)^{2}}+\frac{1}{(c-a)^{2}}]\geq \frac{9(2-k)}{4}$




#606147 $\sum \frac{1}{(1+a)^{3}}\g...

Posted by chardhdmovies on 30-12-2015 - 18:37 in Bất đẳng thức - Cực trị

Kinh khủng quá :)

Sử dụng BĐT Holder ta có:

$4(\sum \frac{1}{(1+a)^3})^2\geq (\sum \frac{1}{(1+a)^2})^3$

Lại có: $\frac{1}{(1+a)^2}+\frac{1}{(1+b)^2}\geq \frac{1}{(a+b)(a+\frac{1}{b})}+\frac{1}{(a+b)(b+\frac{1}{a})}=\frac{1}{ab+1}=\frac{1}{1+\frac{1}{cd}}=\frac{cd}{1+cd}$

Tương tự: $\frac{1}{(1+c)^2}+\frac{1}{(1+d)^2}\geq \frac{1}{1+cd}$

Từ đó suy ra ĐPCM

Thực ra lúc đầu mình cũng làm cách này, cơ mà thấy ko hay lắm nên làm trực tiếp bậc 3 luôn.




#606112 $\sum \frac{1}{(1+a)^{3}}\g...

Posted by chardhdmovies on 30-12-2015 - 12:46 in Bất đẳng thức - Cực trị

cho a,b,c,d là các số thực dương thỏa abcd=1

c/m $\frac{1}{(1+a)^{3}}+\frac{1}{(1+b)^{3}}+\frac{1}{(1+c)^{3}}+\frac{1}{(1+a)^{3}}\geq \frac{1}{2}$

Áp dụng BĐT Holder, ta có: $(1+a)^{3}\leq (1+1)(1+\frac{a\sqrt{a}}{b\sqrt{b}})(1+ab\sqrt{ab})$

Suy ra: $\frac{1}{(1+a)^{3}}\geq \frac{b\sqrt{b}}{2(a\sqrt{a}+a\sqrt{b})(1+ab\sqrt{ab})}$

Tương tự, suy ra: $\frac{1}{(1+b)^{3}}\geq \frac{a\sqrt{a}}{2(a\sqrt{a}+a\sqrt{b})(1+ab\sqrt{ab})}$

Cộng lại ta có: $\frac{1}{(1+a)^{3}}+\frac{1}{(1+b)^{3}}\geq \frac{1}{2(1+ab\sqrt{ab})}$
Thiết lập BĐT tương tự với $c$ và $d$, cộng lại ta có:
$VT\geq \frac{1}{2(1+ab\sqrt{ab})}+\frac{1}{2(1+cd\sqrt{cd})}=\frac{1}{2(1+ab\sqrt{ab})}+\frac{ab\sqrt{ab}}{2(ab\sqrt{ab}+1)}=\frac{1}{2}$

Đẳng thức xảy ra khi và chỉ khi $a=b=c=d=1$




#605714 $P=\frac{a^2}{a^2+b}+\frac{b^2}...

Posted by chardhdmovies on 28-12-2015 - 12:03 in Bất đẳng thức và cực trị

Ta sẽ chứng minh: $P\geq \frac{7}{6}$

$\forall a,b>0,\exists x,y,k>0:a=k\sqrt[3]{\frac{x}{y}},b=k\sqrt[3]{\frac{y}{x}}$. Thay vào ta có:
$P=\frac{kx}{kx+y}+\frac{ky}{ky+x}+\frac{1}{12k^{3}}(\frac{x}{y}+\frac{y}{x})$

$P=(\frac{kx}{kx+y}-\frac{k}{k+1})+(\frac{ky}{ky+x}-\frac{k}{k+1})+\frac{1}{12k^{3}}(\frac{x}{y}+\frac{y}{x}-2)+(\frac{2k}{k+1}+\frac{1}{6k^3})$

$P=(x-y)^2[\frac{k(k-1)}{(k+1)(kx+y)(ky+x)}+\frac{1}{12k^{3}xy}]+\frac{2k}{k+1}+\frac{1}{6k^3}$

Xét hàm thấy $\frac{2k}{k+1}+\frac{1}{6k^3} \geq\frac{7}{6}$
Vậy cần chứng minh:

$\frac{k(k-1)}{(k+1)(kx+y)(ky+x)}+\frac{1}{12k^{3}xy}\geq 0$

Với $k\geq1$, bđt hiển nhiên đúng.

Xét $k<1$, áp dụng bđt Cauchy-Schwarz ta có:

$\frac{k(k-1)}{(k+1)(kx+y)(ky+x)}+\frac{1}{12k^{3}xy}\geq \frac{k(k-1)}{(k+1)^3xy}+\frac{1}{12k^{3}xy}$

Vậy cần chứng minh:
$(k+1)^3+12k^4(k-1)\geq0$

Ta có: $(k+1)^3+12k^4(k-1)>(k+1)^3+12k^2(k-1)\geq(k+1)^3-12\frac{(1-k+k+k)^3}{27}=\frac{5}{9}(k+1)^3>0$

Vậy, GTNN của $P$ là $\frac{7}{6}$, đạt được khi $a=b=1$

Mình sai chỗ này, phải là $1-k$ mới đúng. Lúc đó thì bđt sẽ đúng với $k\leq1$. Còn $k>1$ thì cho $a=1,b \to +\infty $, ta được $P \to \frac{13}{12}$, đề bài có vẻ không ổn.




#605348 $\frac{x}{y+z}+\frac{y}{z+x...

Posted by chardhdmovies on 26-12-2015 - 17:50 in Bất đẳng thức và cực trị

Bạn ơi, x,y,z dương thì làm sao z=0 được

Sorry, mình đọc không kĩ đề, mà thực ra bài này chỉ cần điều kiện $x,y,z \geq0$ và $xy+yz+zx>0$ là đủ rồi.




#605330 $P=\frac{a^2}{a^2+b}+\frac{b^2}...

Posted by chardhdmovies on 26-12-2015 - 15:56 in Bất đẳng thức và cực trị

cho mình hỏi sao lại biết cách đặt như vậy? phương pháp là gì?

Bản chất là đồng bậc bđt thôi bạn, bđt thuần nhất thường dễ làm hơn.




#605296 $\frac{x}{y+z}+\frac{y}{z+x...

Posted by chardhdmovies on 26-12-2015 - 00:37 in Bất đẳng thức và cực trị

Áp dụng Schur, có: $\frac{x}{y+z}+\frac{y}{z+x}+\frac{z}{x+y}+\frac{4xyz}{(x+y)(y+z)(z+x)}\geq 2$

Cần chứng minh: $\frac{3\sqrt[3]{xyz}}{2(x+y+z)} \geq \frac{4xyz}{(x+y)(y+z)(z+x)}$

Cần chứng minh: $3(x+y)(y+z)(z+x)\geq 8\sqrt[3]{x^{2}y^{2}z^{2}}(x+y+z)$

Ta có: $3(x+y)(y+z)(z+x) \geq \frac{8}{3}(x+y+z)(xy+yz+zx) \geq 8(x+y+z)\sqrt[3]{x^{2}y^{2}z^{2}}$

Dấu bằng xảy ra khi và chỉ khi $x=y=z$ hoặc $x=y,z=0$ và hoán vị..




#605290 $P=\frac{a^2}{a^2+b}+\frac{b^2}...

Posted by chardhdmovies on 25-12-2015 - 22:46 in Bất đẳng thức và cực trị

 

Cho a,b dương. Tìm GTNN của biểu thức:

$P=\frac{a^2}{a^2+b}+\frac{b^2}{b^2+a}+\frac{1}{12}(\frac{1}{a^3}+\frac{1}{b^3})$

 

Ta sẽ chứng minh: $P\geq \frac{7}{6}$

$\forall a,b>0,\exists x,y,k>0:a=k\sqrt[3]{\frac{x}{y}},b=k\sqrt[3]{\frac{y}{x}}$. Thay vào ta có:
$P=\frac{kx}{kx+y}+\frac{ky}{ky+x}+\frac{1}{12k^{3}}(\frac{x}{y}+\frac{y}{x})$

$P=(\frac{kx}{kx+y}-\frac{k}{k+1})+(\frac{ky}{ky+x}-\frac{k}{k+1})+\frac{1}{12k^{3}}(\frac{x}{y}+\frac{y}{x}-2)+(\frac{2k}{k+1}+\frac{1}{6k^3})$

$P=(x-y)^2[\frac{k(k-1)}{(k+1)(kx+y)(ky+x)}+\frac{1}{12k^{3}xy}]+\frac{2k}{k+1}+\frac{1}{6k^3}$

Xét hàm thấy $\frac{2k}{k+1}+\frac{1}{6k^3} \geq\frac{7}{6}$
Vậy cần chứng minh:

$\frac{k(k-1)}{(k+1)(kx+y)(ky+x)}+\frac{1}{12k^{3}xy}\geq 0$

Với $k\geq1$, bđt hiển nhiên đúng.

Xét $k<1$, áp dụng bđt Cauchy-Schwarz ta có:

$\frac{k(k-1)}{(k+1)(kx+y)(ky+x)}+\frac{1}{12k^{3}xy}\geq \frac{k(k-1)}{(k+1)^3xy}+\frac{1}{12k^{3}xy}$

Vậy cần chứng minh:
$(k+1)^3+12k^4(k-1)\geq0$

Ta có: $(k+1)^3+12k^4(k-1)>(k+1)^3+12k^2(k-1)\geq(k+1)^3-12\frac{(1-k+k+k)^3}{27}=\frac{5}{9}(k+1)^3>0$

Vậy, GTNN của $P$ là $\frac{7}{6}$, đạt được khi $a=b=1$




#598117 $\frac{a+b}{cosC}+\frac{b+c}...

Posted by chardhdmovies on 13-11-2015 - 12:30 in Bất đẳng thức và cực trị

Cho tam giác nhọn $ABC$ có $AB=c, BC=a, CA=b$, chứng minh:

$\frac{a+b}{cosC}+\frac{b+c}{cosA}+\frac{c+a}{cosB}\geq 4(a+b+c)$




#597812 $\sum \frac{2a}{2a+b+c}\geq 1+\f...

Posted by chardhdmovies on 11-11-2015 - 11:55 in Bất đẳng thức - Cực trị

Thực ra mình làm một bài đến đoạn bđt vừa rồi, áp dụng c-s cộng mẫu ra được cái bđt bị sai. Mình thử vài số thấy đúng nên nghĩ nó đúng  :luoi:




#597718 $\sum \frac{2a}{2a+b+c}\geq 1+\f...

Posted by chardhdmovies on 10-11-2015 - 20:21 in Bất đẳng thức - Cực trị

Cho $a,b,c>0$. C/m:
$\sum \frac{2a}{2a+b+c}\geq 1+\frac{9abc}{2(a+b+c)(ab+bc+ca)}$
P/s: Hy vọng lần này đề không sai




#592964 chọn đội tuyển trường chuyên Nguyễn Du-Đaklak 2015-2016(vòng 2)

Posted by chardhdmovies on 09-10-2015 - 22:40 in Thi HSG cấp Tỉnh, Thành phố. Olympic 30-4. Đề thi và kiểm tra đội tuyển các cấp.

Bài 5 (3 điểm)

Tìm tất cả các nghiệm nguyên dương $(x,y,z)$ của phương trình

$\left | 2^x-y^z \right |=1$

Từ giả thiết ta có: $2^{x}=y^z+1$ hoặc $2^x=y^z-1$. Dễ thấy $y$ lẻ
TH1:  $2^{x}=y^z+1$

Nếu $x=1 \Rightarrow y=1,z=1$

Nếu $y=1 \Rightarrow x=1, z\in\mathbb{N}^{*}$

Nếu $z=1 \Rightarrow y=2^x-1, x\in\mathbb{N}^{*}$

Xét $x>1,y>1,z>1$

Nếu đặt $z=2^{a}t$, $t$ lẻ, $a>0,t>0$, ta có:

$2^{x}=(y^{2^{a}}+1)[(y^{2^{a}})^{t-1}-(y^{2^{a}})^{t-2}+...-y^{2^{a}}+1]$
Giả sử $t>1$, do $y^{2^{a}}$ và $t$ lẻ, lớn hơn $1$ nên $(y^{2^{a}})^{t-1}-(y^{2^{a}})^{t-2}+...-y^{2^{a}}+1$ lẻ và lớn hơn $1$, suy ra phương trình vô nghiệm. Do đó $t=1$.

Suy ra $2^{x}=y^{2^{a}}+1 \Leftrightarrow 2^{x}=(y^{2^{a-1}})^{2}+1$

Dễ thấy phương trình vô nghiệm.

TH2: $2^{x}=y^z-1$. Dễ thấy $y>1$

Nếu $x=1 \Rightarrow y=3, z=1$

Nếu $z=1 \Rightarrow y=2^{x}+1, x\in\mathbb{N}^{*}$

Xét $x>1,y>1,z>1$

Chứng minh tương tự suy ra: $z=2^{b}, b\geq1$

Đặt $y^{2^{b-1}}=c,c>1$, ta có: $2^{x}=(c+1)(c-1)$

Đặt $c+1=2^{e}, c-1=2^{f}, e>f>0$

Suy ra $2^{e-1}-2^{f-1}=1$, suy ra: $f<2$

Nếu $f=0 \Rightarrow 2^{x}=3$

Nếu $f=1 \Rightarrow x=3,y=3,z=2$

Vậy, phương trình có các nghiệm $(3,3,2),(k,2^k+1,1),(n,2^n-1,1),(1,1,l)$ với $\forall k\geq1,n\geq1, l>1$




#592928 chọn đội tuyển trường chuyên Nguyễn Du-Đaklak 2015-2016(vòng 2)

Posted by chardhdmovies on 09-10-2015 - 21:27 in Thi HSG cấp Tỉnh, Thành phố. Olympic 30-4. Đề thi và kiểm tra đội tuyển các cấp.

Bài 6 (4 điểm)

Cho tập hợp $\mathcal{X}=\left \{ \overline{a_6a_5a_4a_3a_2a_1}\mid 9\ge a_6\ge a_5\ge a_4\ge a_3\ge a_2\ge a_1,a_6\neq 0,a_i\in \mathbb{N},(i=\overline{1,6})\right \}$

$a)$ Tập hợp $\left | \mathcal{X} \right |$ có bao nhiêu phần tử $?$

$b)$ Nếu viết tất cả các số của $\mathcal{X}$ thành một dãy tăng.Tính số hạng thứ $2015$ của dãy đó

a)Bổ đề: Số nghiệm nguyên không âm của bất phương trình:
$\sum_{i=1}^{k}x_{i}\leq m$ là $\binom{m+k}{k}$

Trở lại bài toán:

Đặt $a_{i}=\sum_{k=1}^{i}x_{k}$. Ta có:$x_{i}\geq0, \forall i$ và $\sum_{i=1}^{6}x_{i}\leq 9, (1)$

Nếu $a_{6}=0$ thì $a_{6}=a_{5}=a_{4}=a_{3}=a_{2}=a_{1}=0$

Đặt số nghiệm nguyên không âm của $(1)$ là $S$ thì ta có $|X|=S-1$

Áp dụng bổ đề, ta có: $|X|=\binom{9+6}{6}-1=5004$

 

b) Đặt  $T_{i}$ là số các số thuộc $X$ mà $a_{6}=i, i=1,2,..,9$
Khi đó $T_{i}$ là số nghiệm nguyên không âm của bất phương trình $\sum_{k=1}^{5}x_{k}\leq i$

Áp dụng bổ đề, suy ra: $T_{i}=\binom{i+5}{5}$

Ta có: $\sum_{i=1}^{7}T_{i}=1715 < 2015 < \sum_{i=1}^{8}T_{i}=3002 \Rightarrow a_{6}=8$

$2015-1715=300$
Tương tự, đặt $K_{i}$ là số các số thuộc $X$ mà $a_{6}=8, a_{5}=i, i=0,1,2,..,9$

Áp dụng bổ đề, suy ra: $K_{i}=\binom{i+4}{4}$

Ta có: $\sum_{i=0}^{5}K_{i}=252 < 300 < \sum_{i=0}^{6}K_{i}=462 \Rightarrow a_{5}=6$

$300-252=48$

Đặt $L_{i}$ là số các số thuộc $X$ mà $a_{6}=8, a_{5}=6, a_{4}=i, i=0,1,2,..,9$

Áp dụng bổ đề, ta có: $L_{i}=\binom{i+3}{3}$

Ta có: $\sum_{i=0}^{3}L_{i}=35 < 48 < \sum_{i=0}^{4}L_{i}=70 \Rightarrow a_{4}=4$

Suy ra, số hạng thứ $1715+252+35+1=2003$ là $864000$

Suy ra, số hạng thứ $2015$ là $864320$




#590319 chọn đội tuyển trường chuyên Nguyễn Du-Đaklak 2015-2016(vòng 1)

Posted by chardhdmovies on 22-09-2015 - 20:06 in Thi HSG cấp Tỉnh, Thành phố. Olympic 30-4. Đề thi và kiểm tra đội tuyển các cấp.

Bài 6 mình làm bù trừ, không hiểu sao sai đáp số

Đặt $x_1=x-50, z_1=z-50, z_2=z-101$

Ta có $x_1+y+z_1=1965 (1), x_1+y+z_2=1865 (2)$

Gọi $A$ là tập các nghiệm tự nhiên của $(1)$ thỏa $y_1 \leq 100, x_1,z_1 \geq 0$

Gọi $B$ là tập các nghiệm tự nhiên của $(2)$ thỏa $y_1 \leq 100, x_1,z_2 \geq 0$

Với mỗi $y_0$ từ $0-100$ thì số cách chọn $x_1,z_1$ là $C^{1}_{1965-y_0+2-1}$

Suy ra $|A|=1966+1965+....+1866$

Tương tự $|B|=1865+...+1765$

Gọi $C$ là tập các nghiệm thỏa đề thì theo bù trừ, ta có:

$|C|=|A|-|B|=10201$

 

Spoiler

sai chỗ này




#589704 Giải phương trình: $4^{x^{2}}+2^{x}=3^...

Posted by chardhdmovies on 18-09-2015 - 22:04 in Phương trình - Hệ phương trình - Bất phương trình

Giải phương trình: $4^{x^{2}}+2^{x}=3^{x^{2}}+3^{x}$